IAS EXPRESS upsc preparation

Mahatma Gandhi – Biography, Movements, Literary Works

' src=

Mohandas Karamchand Gandhi was a renowned freedom activist and a powerful political leader. He played an important role in India’s struggle for Independence against the British rule of India through non-violent means which ultimately led to Indian independence. He was also known as Father of India, Bapu (endearment for father), and Mahatma (Great Soul). He also improved the lives of India’s poor people and depressed classes. His ideology of truth and non-violence influenced many including Martin Luther and Nelson Mandela. His professions include lawyer (studied law at London, 1888), politician, activist, and writer.

mahatma gandhi essay upsc

This topic of “Mahatma Gandhi – Biography, Movements, Literary Works” is important from the perspective of the UPSC IAS Examination , which falls under General Studies Portion.

Early Life of Mahatma Gandhi:

Mohandas Karamchand Gandhi was born October 2, 1869, in Porbandar, India, to Karamchand Gandhi and Putlibai. At age 13, he married Kasturba Kapadia as part of an arranged marriage. She bore four sons and supported Gandhi’s endeavors until her 1944 death.

In September 1888 at age 18, Gandhi left India alone to study law in London. Gandhi also adopted vegetarianism and joined the London Vegetarian Society , whose intellectual crowd introduced Gandhi to authors Henry David Thoreau and Leo Tolstoy. He was also deeply influenced by the stories of Shravana & Harishchandra, Bhagavad Gita, and Thirukkural (ancient Tamil literature) as they reflected the importance of truth. These books’ concepts set the foundation for his later beliefs.

Gandhi passed the bar on June 10, 1891, and returned to India. For two years, he attempted to practice law but lacked the knowledge of Indian law and the self-confidence necessary to be a trial lawyer. Instead, he took on a year-long case in South Africa.

Express Learning Programme (ELP)

  • Optional Notes
  • Study Hacks
  • Prelims Sureshots (Repeated Topic Compilations)
  • Current Affairs (Newsbits, Editorials & In-depths)
  • Ancient Indian History
  • Medieval Indian History
  • Modern Indian History
  • Post-Independence Indian History
  • World History
  • Art & Culture
  • Geography (World & Indian)
  • Indian Society & Social Justice
  • Indian Polity
  • International Relations
  • Indian Economy
  • Environment 
  • Agriculture
  • Internal Security
  • Disasters & its Management
  • General Science – Biology
  • General Studies (GS) 4 – Ethics
  • Syllabus-wise learning
  • Political Science
  • Anthropology
  • Public Administration

SIGN UP NOW

Contributions of Mahatma Gandhi:

Gandhi’s movement in south africa:.

  • 1893: Gandhi went to South Africa to work as a lawyer. There he had first-hand experience of racial discrimination when he was thrown out of the first-class compartment of the train despite holding the first-class ticket because it was reserved for white people only and no Indian or black was allowed to travel in the first class. This incident had a serious effect on him. He also observed that this type of incident was quite common against his fellow Indians. Therefore, he decided to protest against racial discrimination in South Africa.
  • 1894: Gandhi agrees to extend his stay in South Africa, and helps to establish the Natal Indian Congress . He assumes a prominent and vocal role in the local campaign for Indian rights.
  • 1899: Gandhi forms the Indian Ambulance Corp to provide relief support for British soldiers during the South African War (formerly the Anglo-Boer War).
  • 1901: Goes to India to attend a session of the Indian National Congress. He returns to South Africa the following year.
  • 1902: Gandhi establishes law offices in Johannesburg.
  • 1903: Gandhi begins printing the newspaper, Indian Opinion (later titled Opinion ), in Johannesburg.
  • 1904: Gandhi established The communal Phoenix settlement . Meets Herman Kallenbach (Architect & Gandhi’s associate).
  • 1906: Gandhi gave birth to the idea of Satyagraha . He chaired a meeting in Johannesburg, in which the Indian community was called upon to resist new anti-Asiatic laws through acts of civil disobedience.
  • 1908: Gandhi is arrested for not carrying a passbook. After he was released, he led thousands of Indians in burning their passbooks and registration papers at Johannesburg.
  • 1910: Gandhi and Kallenbach established Tolstoy Farm on the outskirts of Johannesburg. It becomes Gandhi’s base in South Africa.
  • 1913: After the Searle Judgement (under which marriages conducted under Indian law were declared invalid), many Indian women join the Satyagraha.
  • 1914: Suspends Satyagraha, in return for the abolition of taxes, recognition of Indian marriages, and the freedom of movement of Indians.
  • Movement in South Africa transformed Gandhi into a new person and it played an important role in the evolution of Satyagraha which was used in upcoming freedom struggles in India.

Gandhi’s Movement in India:

  • 1915: Gandhi returned to India and joined the Indian National Congress with Gopal Krishna Gokhale as his mentor.
  • 1916: He established the Sabarmati Ashram at Ahmedabad so that his followers could learn and practice the truth and non-violence.
  • 1917: Champaran satyagraha –  marks Mahatma Gandhi’s first successful application of his method of ‘satyagraha’ in India. The indigo cultivators of Champaran, Bihar were greatly exploited by the planters who were largely European. The cultivators were bound by law to grow indigo on 3/20 th (the tinkathia system) of the total area of their land and sell it to the planters at the price fixed by the latter. As a result of this system, the poor peasants were greatly distressed and approached Gandhi in order to lead their struggle against the planters. Gandhi accepted their request but was banned from entering the district by district authority orders. In protest, Gandhi did satyagraha. As a result of his campaign, an enquiry was conducted into the conditions of the peasants. This went a long way in giving the indigo cultivators some relief.
  • Ahmedabad Mill Strike : In February 1918, there was a conflict between mill owners and workers of the Gujrat Mill. Gandhiji organised satyagraha in favour of cotton mill workers in 1918 at Ahmedabad. It was one of the formative events in the political career of Mahatma Gandhi. 
  • Kheda Satyagraha of 1918: Gandhi organised this movement to support peasants of the Kheda district, Gujarat. The people of Kheda were unable to pay the high taxes levied by the British due to crop failure and a plague epidemic.
  • Young India was a weekly paper or journal in English started by Mahatma Gandhi. It was published from 1919 to 1931. Through this work, he desired to popularise India’s demand for self-government or Swaraj. He used Young India to spread his unique ideology and thoughts regarding the use of nonviolence in organising movements and to urge readers to consider, organise, and plan for India’s eventual independence from Britain. 
  • Navajivan (a new life) was weekly newspaper published by Gandhi, in Gujarati , from 1919 to 1931, from Ahmedabad.
  • 1920-22: Non-Cooperation Movement – Gandhi launched this mass movement which involved participation from the nationalists as well as the public.  Factors that led to the formation of this movement were the oppressive policies of Britishers such as the Rowlatt Act and the Jallianwala Bagh incident in Amritsar. The campaign involves Indians revoking their cooperation from the British government, with the aim of inducing the British to grant self-governance (swaraj). The British government arrested Gandhi and sentenced him to six years in jail for sedition. Gandhi withdrew the non-cooperation movement after the Chauri-Chaura Incident . After the non-cooperation movement ended, Gandhi withdrew from the political platform and focused on his social reform work.
  • 1929: Navajivan Trust  is a  publishing house  based in  Ahmedabad ,  India . It was founded by  Mahatma Gandhi in 1929  and has published more than 800 titles in  English ,  Gujarati ,  H i ndi and other languages to date. The objective of Navajivan Trust was to propagate peaceful means for the attainment of  Hind Swaraj  (Swaraj for India).
  • 1930: Civil Disobedience Movement – The Simon Commission came to India in 1928. It was to look into the functions of the constitutional system in India. As there was no Indian member in it, it was boycotted by all political parties in the country. Later, in 1929, the Congress under the leadership of Nehru declared ‘Poorna Swaraj’ as its main goal. As nationalist feelings began to rise, Gandhi sent a letter containing eleven demands to Lord Irwin in 1930 asking him to accept it. When he refused, Gandhi launched the Civil Disobedience Movement. Civil disobedience implies the active, professed refusal of a citizen to obey certain laws, orders, and demands of a government. In the year 1930, Gandhi started this movement (Dandi) by violating the salt law. The movement was discontinued following the Gandhi Irwin Pact of 1931 .
  • Communal Award – Ramsay Macdonald announced communal award due to repeated failure of communities to decide on the proportion of representation. The award provided for a separate electorate for depressed classes. Gandhi condemned this award as it was based on Britishers’ Divide and Rule Policy and it will destroy Hinduism. He led a fast unto death which ultimately resulted in Poona Pact between Congress and depressed classes headed by B.R. Ambedkar . Poona pact provides for reservation of depressed classes in Hindu joint electorate.
  • As a result of the communal awards and Poona pact, Gandhi dedicated himself to the upliftment of depressed classes and untouchables. Gandhi founded Harijan Sevak Sangh (All India Anti-Untouchability League) in 1932. He also started a journal named Harijan which means “People of God”.
  • 1934 : Gandhi resigned from the Congress party membership as he did not agree with the party’s position on different issues.
  • 1936: Gandhi returned to active politics in 1936 with the Lucknow session of Congress where Jawaharlal Nehru was the president.
  • 1938 : Gandhi and  Subhash Chandra Bose ’s principles clashed during the Tripuri session which led to the Tripuri crisis in the Indian National Congress.
  • 1942: Quit India Movement – The Quit India Movement was launched by Mahatma Gandhi on 8 August 1942 at the Bombay session of the All India Congress Committee (AICC). Also known as the Bharat Chhodo Andolan , this movement was a mass civil disobedience that took place in the country. Gandhi demanded that the British must leave India immediately or face dire consequences. As a part of this movement, a call for mass agitation was followed by violence that took over the country after which Indian National Congress leaders were arrested. The main reason for the Quit India Movement to begin was because the Britishers were planning to drag the country into World War II without consent to fight on behalf of the United Kingdom (UK). During that time, more than 87,000 Indian soldiers were martyred in World War II including people from Pakistan, Nepal, and Bangladesh. Also, the Cripps Mission which was led by Sir Stafford Cripps, a member of the War Cabinet in March 1942, made an attempt to secure India’s cooperation in World War II. Following this, Cripps was sent to India to discuss and support the British Government’s Draft Declaration with Indian leaders. Moreover, the declaration also granted India Dominion Status after the war. To which, Congress denied discussing any terms unless given total freedom.
  • 1947: India wins independence. Gandhi fasts as penance for inter-communal violence following the partition of India and Pakistan.

Prelims Sureshots – Most Probable Topics for UPSC Prelims

A Compilation of the Most Probable Topics for UPSC Prelims, including Schemes, Freedom Fighters, Judgments, Acts, National Parks, Government Agencies, Space Missions, and more. Get a guaranteed 120+ marks!

Gandhian Ideologies:

Gandhi developed a set of religious and social ideas first during his period in South Africa from 1893 to 1914 and later in India. He developed these ideologies from various inspirational sources including Bhagavad Geeta, Jainism, Buddhism, Bible, Gopal Krishna Gokhale, Tolstoy (His book The Kingdom of God is within you had a deep influence on Gandhi), John Ruskin (Gandhi paraphrased his book Unto the Last as Sarvodaya ), among others. These ideologies have been further developed by later Gandhians most notably, in India by Vinoba Bhave and Jayaprakash Narayan , outside of India by Martin Luther King Jr., Nelson Mandela, and others. Major Gandhian ideologies are as follows.

Truth and Non-Violence: 

  • They are the twin cardinal principles of Gandhian thoughts.
  • Relative truth of truthfulness in word and deed.
  • Absolute truth – the ultimate reality. This ultimate truth is God (as God is also Truth). Morality – the moral laws and code – its basis.
  • Nonviolence is Active love, that is, the polar opposite of violence, in every sense. Nonviolence or love is regarded as the highest law of humankind.

Satyagraha:

  • It is a method of securing rights through nonviolent action, that is, through personal suffering rather than inflicting injury on others.
  • It means the exercise of the purest soul-force against all injustice, oppression, and exploitation.
  • The origin of Satyagraha can be found in the Upanishads, and the teachings of Buddha, Mahavira, and other greats including Tolstoy and Ruskin.
  • The term Sarvodaya means ‘Universal Uplift’ or ‘Progress of All’.
  • It was first coined by Gandhiji as the title of his translation of John Ruskin’s book on political economy, Unto the Last .
  • Although the word swaraj means self-rule, Gandhiji gave it the content of an integral revolution that encompasses all spheres of life.
  • The sum total of the swaraj (self-rule) of individuals.
  • Freedom for the meanest of his countrymen. 
  • Much more than freedom from all restraints, it is self-rule and self-restraint and could be equated with moksha or salvation.
  • He envisaged Ram Rajya where the swaraj is achieved for the people by the people.
  • The word swadeshi is a conjunction of two Sanskrit words – ‘Swa’ means self or own and ‘desh’ means country. So Swadesh literally means one’s own country. But it can be loosely translated in most contexts as self-sufficiency.
  • It is the focus on acting within one’s own community, both politically and economically.
  • It is the interdependence of Community and Self-sufficiency.
  • Gandhi believed this would lead to independence (swaraj) as the British control of India was rooted in control of her indigenous industries.
  • Swadeshi was represented by the charkha or the spinning wheel, The “center of the solar system” of Mahatma Gandhi’s constructive program.

Trusteeship:

  • It is a socio-economic philosophy propounded by Gandhiji.
  • It provides a means by which the wealthy people would be the trustees of Trusts that looked after the welfare of the people in general.
  • Gandhi believed that Education needs to be a lifelong experience.
  • He developed a scheme on education named Nayee Taleem.
  • He gave priority to vocational education, the idea of ‘earn & learn’, and the learnings like social forestry, nursing, home science, handicrafts, etc.

Death of Gandhi:

On 30th January 1948, Gandhi was on his way to address a prayer meeting in the Birla House New Delhi when a Hindu fanatic named Nathuram Godse fired three bullets into his chest from close range killing him instantly.

Legacy of Gandhi:

Gandhi always held on to non-violence and simple living throughout his life, in his principles, practices, and beliefs. He influenced several great leaders and the country respectfully addresses him as the father of the nation or Bapu. Rabindranath Tagore is said to have given the title of Mahatma to Gandhi. It was Netaji Subhash Chandra Bose who first addressed him as the Father of the Nation. Many great world leaders like Nelson Mandela followed Gandhiji’s teachings and way of life. Therefore, his impact on the world is still very high.

Literary Works of Gandhi:

  • Hind Swarajya (1909),
  • Indian Home Rule (1910),
  • Sermon on the Sea (1924 – the American edition of Hind Swaraj),
  • Dakshina Africana Satyagrahano Itihasa / Satyagraha in South Africa (1924-25),
  • Satyana Prayogo Athava Atmakatha / An Autobiography: The Story of My Experiments With Truth (1924-25),
  • Mangalaprabhata (1930),
  • India’s Case for Swaraj (1931),
  • Songs from Prison: Translations of Indian Lyrics Made in Jail (1934),
  • The Indian States’ Problem (1941),
  • The Good life (1943),
  • Gandhi Against Fascism (1944),
  • From Yeravda Mandir: Ashram Observances (1945),
  • Conquest of Self (1946),
  • Women and Social Injustice (1947),
  • Self-restraint v. Self-Indulgence (1947),
  • Gandhigrams (1947). 

Related Posts

Tej Bahadur Sapru mind map

Sardar Vallabhai Patel – Biography and Contributions

periyar e v ramasamy upsc essay notes mindmap

Periyar E. V. Ramasamy – Biography, Contributions, Literary Works...

Sahajanand Saraswati - Biography, Contributions, Literary Works

Sahajanand Saraswati – Biography, Contributions, Literary Works

guest

your accommodated content is really worthful

Santhosh

Thank you. Keep reading for more such content.

There was a problem reporting this post.

Block Member?

Please confirm you want to block this member.

You will no longer be able to:

  • See blocked member's posts
  • Mention this member in posts

Please allow a few minutes for this process to complete.

Express LMS for UPSC banner

  • Skip to main content

India’s Largest Career Transformation Portal

Essay on Mahatma Gandhi for Students in English [500+ Words]

December 10, 2020 by Sandeep

Essay on Mahatma Gandhi:  The entire life of Mahatma Gandhi belonged to truthiness, vegetarianism, simplicity, non-violence, and faith in god. During his life, he had been an inspiration to many national and international leaders. He trusted in truth and never took the assistance of fakeness or lie to accomplish their tasks. In India, a public holiday is declared on 2nd October ( Gandhi Jayanti ) to honour him. Also, to mark his sacrifice for his nation, his image has engraved on the Indian currency notes. Below we have provided Mahatma Gandhi Essay in English, written in easy and simple words for class 5, 6, 7, 8, 9 and 10 school students.

Essay on Mahatma Gandhi in English

Mahatma Gandhi was a man of power, honesty, sacrifices, and simplicity. He was a patriotic personality. His life was unbelievable. He never liked others’ appreciation. His efforts to protect India are unparalleled. Only with his efforts, the British left India in 1947. Mohandas Gandhi is another name of Mahatma Gandhi. Even he is also famous as ‘Father of Nation.’ He was born on 2nd October in the year 1869 in the state of Porbandar, Gujarat. He was the youngest among the three sons of Karamchand Gandhi.

Karamchand Gandhi and Putlibai Gandhi were his parents. His mother was the fourth wife of his father, Karamchand Gandhi. His father became the prime minister of states like Rajkot, Vankaner, and Porbandar. His parents have three sons. When Mahatma Gandhi became 13 in age, he got married to Kasturba Makhanji. After his marriage, he completed his education. Then, he went to London to pursue a degree in law. After this, he practiced as a lawyer. He also started to read the scriptures of Bhagavad Gita that had an immense influence on his life.

During his childhood, he was massively influenced by the tales of Harischandra and Shravana that taught him about the significance of affection and truth. Moreover, his parents raise him as a vegetarian; however, during his stay in London, he started eating meat. Once Gandhiji came back to India after finishing his study in law, he started finding a job as a lawyer. He had to do a lot of struggle to find the right job. Lastly, he got an invitation from South Africa by a wealthy merchant to work as a lawyer. That proposal was a turning point in his life.

During his job in South Africa, he witnessed racial discrimination. Once he faced several humiliations, he decided to raise his voice. Then, he became an activist to find cases in favor of Indians. At that time, he also established the Natal Indian Congress in 1894. Besides, he was also influenced by Satyagraha, which means the devotion of truth. Even, he started non-violent protests in 1906. Also, he stood for civil rights in favor of South Africa. Movements started by Mahatma Gandhi in favour of India are

Mahatma Gandhi Essay

Khilafat Movement: This movement was run from 1919-1924. It was a revolt by the nationalists and the Indian Muslims. This movement was started to pressure the British Government to preserve the authority of the Ottoman Sultan as the Islamic Caliph. The leadership of this movement comprised Muhammad Ali and Shaukat Ali, who had been newspaper editors in Delhi in those days. Maulana Mahmud ul-Hasan, Abdul Kalam Azad, Maulana Abdul Bari were also some of the leaders of this movement. During this movement, Mahatma Gandhi even got the support of the Muslims to fight against the British to make Indian independence. This movement was crucial as it reflected Hindu-Muslim cooperation. Finally, in the year 1921, this movement was suppressed by the British Government.

Non-cooperation Movement: To provide the Indian a self-government, Mahatma Gandhi took a step where he organized a non-cooperation movement from 1920-1922. This non-violent movement was also known as Swaraj. The commencement of this non-cooperation movement was the outcry over the Jallianwalla Bagh massacre in the year 1919. In Jallianwalla Bagh (in Amritsar, Punjab), the British government had killed hundreds of Indians who were gathered for peaceful assembly in a lawn. The complete garden becomes a ground of cruel massacre. General Dyer, who was responsible for that act, was not taken to task. Being a part of this movement, Indians boycott the Indian government and also refuses to pay taxes.

Salt Satyagraha: When the British government in India started levying excess taxes or excise duty on salt, then the Mahatma Gandhi began to historic Salt Satyagraha to break this law. For this, Gandhi Ji marched to the coastal village of Dandi to break the salt law. During his march, he used to visit different communities to teach about the need for social reforms. This march was last long for 24 days. After viewing this, the British government arrested the people from Indian National Congress who were involved in this march. When the government got pressurized for the arrests as of several outbreaks of civil disobedience, then they released the arrested leaders.

Quit India Movement: Bharat Chodo Andolan is another name of the Quit India Movement that was launched on 8th August 1942 by the All India Congress Committee. As the name suggests, this movement was started to end up the British rule from India. Mahatma Gandhi started this movement at Gowalia Tank Maidan, where he taught the Indians to “Do or Die.” Even those leaders who got arrested during this movement which took a step back. The protest was a mix of violent and non-violent demonstrations. The fact is, the British did not want to leave India in pressure or with this movement. Before leaving, they wanted the war only. Finally, in 1945, the British revealed that they would send a planned withdrawal from the country.

Mahatma Gandhi was not in favor of dividing India. But he was agreed to keep harmony. Moreover, Gandhiji initiated his fast to make sure of the harmony between the two countries. Besides, Pakistan was granted with payment according to the Partition Council agreement. On 30th January 1948, with the bullets of Nathuram Godse, Mahatma Gandhi succumbed. Nathuram Godse was from the group of Hindu radicals who trusted that India became weakened as of the partition payment made to Pakistan.

  • Skip to primary navigation
  • Skip to main content
  • Skip to primary sidebar

UPSC Coaching, Study Materials, and Mock Exams

Enroll in ClearIAS UPSC Coaching Join Now Log In

Call us: +91-9605741000

Gandhian Philosophy

Last updated on September 14, 2023 by ClearIAS Team

gandhian philosophy

Mohandas Karamchand Gandhi , popularly known as Mahatma Gandhi was a unique personality.

What made him special?

What were his views? What is the importance of Gandhian Philosophy? How it is relevant in modern times?

Table of Contents

What is Gandhian philosophy?

Gandhian Philosophy is the religious and social ideas adopted and developed by Mahatma Gandhi.

Gandhian Philosophy exists on several planes – spiritual or religious, moral, political, economic, social, individual, and collective.

Gandhian Philosophy emphasizes not utopian idealism, but practical idealism.

ClearIAS UPSC Coaching

Roots of Gandhian Philosophy

Gandhi was born in India in 1869. From 1893 to 1914 he lived in South Africa. Later, he emerged as the face of Indian freedom movement .

The base of Gandhian philosophy is the spiritual or religious component. He was a firm believer in God.

He was also a believer in humanity.

He believed that human nature is good. He considered all people are capable of moral development.

Gandhiji developed these ideologies from various inspirational sources such as  Bhagavad Geeta ,  Jainism, Buddhism, Bible, Gopal Krishna Gokhale, Tolstoy, and John Ruskin among others.

Leo Tolstoy’s book ‘ The Kingdom of God is within you ” had a deep influence on Mahatma Gandhi. The same was the case with John Ruskin’s ‘Unto This Last’.

Gandhiji paraphrased Ruskin’s book ‘Unto This Last’ as ‘Sarvodaya’, which meant the upliftment of all.

ClearIAS UPSC Prelims Test Series

The goal of Gandhi was to change society and each individual using the values of honesty and nonviolence.

These ideas were further developed by many followers of Gandhi who proudly called themselves “Gandhians”.

Those who actively followed Gandhian principles in life include Vinoba Bhave,  Jayaprakash Narayan, and Martin Luther King Jr.

Also read: Acharya Vinoba Bhave

Important Gandhian Philosophies

1. truth and non-violence.

These are generally considered to be the two key ingredients of Gandhian thought.

It is possible to pursue one without the other. Thus, seeking the truth can be done violently. Nations enter conflicts assuming they are on the side of truth or that the truth is on their side.

Those who are more sensitive and think the truth is on their side demand that a just war should be fought instead of one that is avoided at all costs.

The most sensitive were the pacifists among them. By avoiding violence altogether. But it could be argued that in doing so they have gone too far and abandoned truth, especially when interpreted as justice.

Even Mahatma Gandhi argued that although he was opposed to war, the two parties engaging in it may not stand on the same plane: the cause of one side could be more just than the other so that even a nonviolent person might wish to extend his or her moral support to one side rather than to the other.

Thus just as it is possible to pursue truth without being nonviolent, it is also possible to pursue nonviolence without pursuing truth.

It could be proposed that such a disjunction between the two runs the risk of cowardice being mistaken for, or masquerading as nonviolence.

The point becomes clear if we take the word “truth” to denote the “right” thing to do in a morally charged situation.

Gandhi’s opinion on the Non-violent Way

By using a non-violent approach, we aim to eliminate capitalism, not the capitalist. We encourage the investor to think of himself as a trustee for people who rely on him to create, hold onto, and grow his capital.

The worker is not required to wait for his conversion. Work is power if money is. Both are depending upon one another.

The moment the worker recognizes his potential, he is in a position to stop being the capitalist’s slave and start sharing in his success.

If he aims at becoming the sole owner, he will most likely be killing the goose that lays the golden eggs.

No need for me to worry that someone will replace me if I refuse to cooperate. Because I hope to persuade my co-workers to oppose my employer’s wrongdoing, This method of mass worker education is undoubtedly a slow one, but as it is also the most reliable, it must also be the fastest.

It is simple to show in the end that the worker is right and that no human being is so flawless as to merit his eliminating the person whom he mistakenly believes to be completely evil.

2. Satyagraha

The concept was introduced in the early 20th century by Mahatma Gandhi and designated a determined but nonviolent resistance to evil.

The supreme idea of truth naturally leads to the Gandhian ideology of Satyagraha. Protecting the standards and tenets of truth is essential if it is the ultimate reality. God, who is the utmost truth and reality, requires a votary who is completely compassionate and unselfish.

Gandhi’s Satyagraha became a major tool in the Indian struggle against British imperialism and has since been adopted by protest groups in other countries.

The ancient Indian philosophy of ahimsa, or “non-injury,” which is rigorously practised by Jains, many of whom reside in Gujarat, where Gandhi was raised, is the inspiration behind Satyagraha.

Gandhi also drew inspiration from the works of Leo Tolstoy and Henry David Thoreau, the Bible, and the Bhagavad Gita, on which he produced a commentary, to modernize the notion of ahimsa and give it broad political implications as Satyagraha.

Gandhi first conceived Satyagraha in 1906 in response to a law discriminating against Asians that was passed by the British colonial government of the Transvaal in South Africa.

In 1917, the Champaran district, which produced indigo, hosted the first Satyagraha campaign in India. Fasting and economic boycotts were used as Satyagraha tactics in India over the ensuing years, up until the British left in 1947.

Since Satyagraha depends on the opponent, who is the embodiment of evil, upholding a high standard of ethical conduct, and demands an unreasonably high level of commitment from those working for social change, critics of the movement have asserted that it is unrealistic and incapable of achieving universal success.

These arguments have been made both during Gandhi’s lifetime and since.

However, Satyagraha left a lasting legacy in South Asia and was a key component of the civil rights movement headed by Martin Luther King Jr. in the United States.

3. Sarvodaya

The word “Sarvodaya” means “Universal Uplift” or “Progress of All.” The phrase was first used by Mohandas Gandhi to describe the aim of his political philosophy in his 1908 translation of “Unto This Last,” a work by John Ruskin on political economy.

Later Gandhians adopted the phrase as a moniker for the social movement in post-independence India that worked to ensure that self-determination and equality reached all strata of Indian society, including the Indian nonviolence leader Vinoba Bhave.

Objects of the Sarvodaya Movement

The Sarvodaya Movement has as its target the establishment of a whole network of such self-supporting village communities.

 Family ties, which are currently restricted to blood groups, will be extended to include the entire village, erasing any disparities based on race, creed, caste, language, and other factors.

The planning of agriculture will ensure that there is enough food for everyone. Up until everyone in the hamlet has a job, the industry will operate on a cottage basis.

Village Council, a body that represents the entire village, will be responsible for determining the requirements of the community.

Principles of the Sarvodaya

  • There is no centralized authority, and there is a political and economic atmosphere in the villages.
  •  The spirit of love, fraternity, truth, nonviolence, and self-sacrifice will permeate all people. The foundation of society will be nonviolence.
  • There will be no party system and majority rule and society will be free from the evil of the tyranny of the majority.
  • Socialist in the truest sense, the Sarvodaya society. The same ethical, social, and financial standards will apply to all calls. The greatest potential for development exists within each person’s personality.
  •  Sarvodaya society is based on equality and liberty. There is no room in it for unwholesome competition, exploitation, and class hatred.
  • Sarvodaya is a symbol of universal progress. Every person should work independently and adhere to the concept of non-possession. The goal of: from each according to his effort and each according to his needs will then be achievable.
  • There won’t be any private property, which serves as a weapon of exploitation and a breeding ground for prejudice and hatred. Similar to how the profit motive will vanish, rent and interest will also disappear.
  • The Sarvodaya Movement is based on Truth, nonviolence, and Self-denial.
  • The Sarvodaya Movement makes an earnest and audacious effort to foster the environment required to unite such people with steadfast trust in the Welfare of All.
  • The benefit to the person would be little. Each quality’s growth is dependent on every other quality. If every quality were somewhat enhanced, the person would benefit more.

4. Swadeshi

The combination of two Sanskrit terms yields the English word “Swadeshi,” which has Sanskrit roots. Swa and desh both refer to one’s own or one’s nation.

Swadesh, therefore, refers to one’s homeland. The adjectival version of the word swadeshi, which means “of one’s own country,” can be loosely translated as “self-sufficiency” in most settings.

The message of the Charkha

Gandhiji asserts that the Charkha has the distinction of being able to address the issue of economic distress in a way that is most logical, straightforward, affordable, and professional.

It stands for both the wealth and freedom of the country. It represents commercial peace rather than commercial strife.

The spinning-meaning wheel is considerably bigger than its diameter. Simple life, helping others, living without hurting others, and forging an unbreakable tie between the wealthy and the poor, capital and labour, and the prince and the peasant are all part of its teachings. Naturally, the bigger lesson applies to everyone.

5. Trusteeship

Trusteeship is a key component of Gandhian economics that could be called the nonviolent equivalent of ownership.

The idea was taken from English law by Gandhi. It denotes that one is a trustee rather than the owner of their belongings, including eventually their skills or abilities.

All must be used for the greater good of society, which includes one’s own welfare in the end. In this system, material possessions do not serve as status symbols that increase our sense of self-worth.

Trusteeship is a successful strategy for reducing excessive consumption. The economy might be rebalanced under trusteeship and put its focus back on pressing needs.

Gandhi believed that possessing more than one need necessarily entails robbing others of their requirements. There is enough on the earth to meet everyone’s needs, but not enough to satisfy everyone’s greed, he said.

The relevance of Gandhian Philosophy in Modern Times

Make in India is a manifestation of Gandhi’s ideals of self-sufficiency.

Gandhi’s philosophy of inclusive growth is fundamental to the building of a resurgent rural India.

He believed in “production by the masses” rather than in mass production, a distinctive feature of the industrial revolution.

Environment

 Gandhi warned the country of unrestricted industrialism and exploitation of nature for human greed.

The results of not adhering to Gandhian environmentalism are serious environmental damage and non-sustainable development.

Administration

By emulating Gandhi’s values of non-violence and Satyagraha, internal matters such as insurgency issues in Kashmir, central India, or the North-Eastern states might be handled much better.

International

Even India’s foreign policy is founded on peaceful coexistence, and this is seen in the fact that India does not engage in aggression initially, even when security threats mount.

To conclude we can say that most of the teachings of Mahatma Gandhi hold relevance even in today’s world.

Seven Social Sins by Mahatma Gandhi

Seven Social Sins by Mahatma Gandhi were first published in his newspaper Young India in 1925. Those are a comprehensive list of behaviours that cause serious harm to society.

1. Wealth without Work

2. Pleasure without Conscience

3. Knowledge without Character

4. Commerce (Business) Without Morality (Ethics)

5. Science without Humanity

6. Religion without Sacrifice

7. Politics without Principle

  • Wealth without Work:  It depicts making wealth by unfair means, by taking shortcuts. Examples: Black Money, Tax evasion, scams, insider trading, etc.
  • Pleasure without Conscience:  Earning happiness at the expense of others is equivalent to sin. Selfishness compels a person to disregard the needs of others. Without moral justification, it would promote bad practices. Additionally, it would lead to a rise in drug and alcohol abuse as well as mindless shopping.
  • Knowledge without character:  A person with character possesses attributes of honesty and integrity. A person who commits this vice may end up like Osama Bin Laden, while a person with moral character may end up like Swami Vivekananda.
  • Business without morality:  One particular segment of the community would end up being overworked at the expense of another. Social friction and community conflict would rise as a result of this. Examples of this sin include unsafe working conditions, adulteration, and lack of security.
  • Science without humanity:  The huge pharmaceutical companies keep prescription prices high, making them unaffordable for the poor and those in need. If nuclear power is utilized to generate electricity, that is great, but using it to destroy nations like Hiroshima and Nagasaki by bombing is utterly immoral.
  • Religion without sacrifice:  Today’s religion consists merely of rituals and activities. Sin is when we fail to live out the moral precepts of brotherhood, compassion, and affection.
  • Politics without principle:  Criminalization of politics, unaccounted money, and use of muscle power shows politics without principle.

To read more about Mahatma Gandhi’s biography click here.

Article Written by: Remya

Print Friendly, PDF & Email

Aim IAS, IPS, or IFS?

ClearIAS UPSC Coaching

About ClearIAS Team

ClearIAS is one of the most trusted learning platforms in India for UPSC preparation. Around 1 million aspirants learn from the ClearIAS every month.

Our courses and training methods are different from traditional coaching. We give special emphasis on smart work and personal mentorship. Many UPSC toppers thank ClearIAS for our role in their success.

Download the ClearIAS mobile apps now to supplement your self-study efforts with ClearIAS smart-study training.

Reader Interactions

Leave a reply cancel reply.

Your email address will not be published. Required fields are marked *

Don’t lose out without playing the right game!

Follow the ClearIAS Prelims cum Mains (PCM) Integrated Approach.

Join ClearIAS PCM Course Now

UPSC Online Preparation

  • Union Public Service Commission (UPSC)
  • Indian Administrative Service (IAS)
  • Indian Police Service (IPS)
  • IAS Exam Eligibility
  • UPSC Free Study Materials
  • UPSC Exam Guidance
  • UPSC Prelims Test Series
  • UPSC Syllabus
  • UPSC Online
  • UPSC Prelims
  • UPSC Interview
  • UPSC Toppers
  • UPSC Previous Year Qns
  • UPSC Age Calculator
  • UPSC Calendar 2024
  • About ClearIAS
  • ClearIAS Programs
  • ClearIAS Fee Structure
  • IAS Coaching
  • UPSC Coaching
  • UPSC Online Coaching
  • ClearIAS Blog
  • Important Updates
  • Announcements
  • Book Review
  • ClearIAS App
  • Work with us
  • Advertise with us
  • Privacy Policy
  • Terms and Conditions
  • Talk to Your Mentor

Featured on

ClearIAS Featured in The Hindu

and many more...

Reading Comprehension: Online Workshop

Thank You 🙌

mahatma gandhi essay upsc

Learn the short-cut tips nobody knows about solving passages.

Take clearias mock exams: analyse your progress.

ClearIAS Course Image

Analyse Your Performance and Track Your All-India Ranking

Ias/ips/ifs online coaching: target cse 2025.

ClearIAS Course Image

Are you struggling to finish the UPSC CSE syllabus without proper guidance?

  • UPSC IAS Exam Pattern
  • UPSC IAS Prelims
  • UPSC IAS Mains
  • UPSC IAS Interview
  • UPSC IAS Optionals
  • UPSC Notification
  • UPSC Eligibility Criteria
  • UPSC Online
  • UPSC Admit Card
  • UPSC Results
  • UPSC Cut-Off
  • UPSC Calendar
  • Documents Required for UPSC IAS Exam
  • UPSC IAS Prelims Syllabus
  • General Studies 1
  • General Studies 2
  • General Studies 3
  • General Studies 4
  • UPSC IAS Interview Syllabus
  • UPSC IAS Optional Syllabus

mahatma gandhi essay upsc

UPSC Essay Notes – Important Personalities – Mahatma Gandhi

In the annals of Indian history, few figures loom as large or command as much reverence as Mahatma Gandhi. His name is synonymous with the ideals of nonviolence, civil disobedience, and social justice. Born Mohandas Karamchand Gandhi on October 2, 1869, in Porbandar, Gujarat, he would later be affectionately known as the Mahatma, meaning “Great Soul.” Through his unwavering commitment to Satyagraha, or the pursuit of truth through nonviolent resistance, Gandhi led India to independence from British rule, leaving an indelible mark on his nation and the world at large. His life and teachings continue to inspire generations, serving as a beacon of hope and a timeless reminder of the power of peaceful protest and moral courage.

Table of Contents

1. Nai Talim

In 1937, Mahatma Gandhi seeded an important idea to revamp the education system, at a conference in Wardha,B Maharashtra through Nai Talim. Gandhi felt a need to nullify what education at the time of British Raj was trying to create – distinctions between knowledge and work, teaching and learning, among others. He also considered education as a medium to combat the dominant societal malice of ‘untouchability’ associated with caste-based vocations, such as spinning, weaving, basket-making, leather-work, and pottery.

He envisioned his scheme of education as the one that would lead to silent social revolution by eradicating poisoned relationships between classes. He was a believer in the power of education and wanted education to be accessible to all. This, he thought, would then help changing the dominant mindset that considered:

  • manual work as inferior to mental work
  • education to be a prerogative of upper castes alone

“By education, I mean an overall, all around drawing out the best in child and man, in body, mind, and spirit,” Gandhi said. He wanted an education system in which education and labour are complementary and felt this move in turn would help in eliminating unnatural division between ‘haves’ and ‘have-nots’, rural-urban divide through equitable balances. The dignity of labour and skill-based learning were the hallmarks of the Gandhian approach.

Swaraj for Gandhi was not freedom from the British. He said, clearly, “Real swaraj will come, not by the acquisition of authority by a few, but by the acquisition of the capacity by all to resist authority when it is abused. In other words, swaraj is to be attained by educating the masses to a sense of their capacity to regulate and control authority.” Although the word swaraj means self-rule, Gandhi gave it the content of an integral revolution that encompasses all spheres of life.

  • At the individual level, swaraj is vitally connected with the capacity for dispassionate self-assessment, ceaseless self-purification, and growing swadeshi or self-reliance.
  • Politically swaraj is self-government and not good government (for Gandhi, good government is no substitute for self-government) and it means continuous effort to be independent of government control, whether it is foreign government or whether it is national. In other words, it is sovereignty of the people based on pure moral authority.
  • Economically, poorna swaraj means full economic freedom for the toiling millions.

For Gandhi, swaraj of the people meant the sum total of the swaraj (self-rule) of individuals and so he clarified that for him swaraj meant freedom for the meanest of his countrymen. And in its fullest sense, swaraj is much more than freedom from all restraints, it is self-rule, self-restraint and could be equated with moksha or salvation.

Swaraj means vast organising ability, penetration into the villages solely for the services of the villagers; in other words, it means national education i.e., education of the masses. And in the Gandhian discourse, education of the masses means conscientization, mobilisation and empowerment, making people capable and determined to stand up to the powers that be.

3. Sarvodaya

Sarvodaya is a term meaning ‘Universal Uplift’ or ‘Progress of All’. The term was first coined by Gandhi as the title of his 1908 translation of John Ruskin’s tract on political economy, “Unto This Last”, and Gandhi came to use the term for the ideal of his own political philosophy. Later Gandhian, like the Indian activist Vinoba Bhave, embraced the term as a name for the social movement in post-independence India which strove to ensure that self-determination and equality reached all strata of Indian society.

Inspirations from Ruskin’s Book:

  • That the good of the individual is contained in the good of all.
  • A lawyer’s work has the same value as the barber’s in as much as all have the same right to earn their livelihood from their work.
  • That is a life of labour, i.e., the life of the tiller of the soil and the handicraftsman is the life worth living.

Objects of Sarvodaya Movement:

The Sarvodaya Movement has as its target the establishment of a whole network of such self-supporting village communities .

  • The family relationships which are confined at present to the blood group will be extended to cover the whole village where distinctions based on race, creed, caste, language and so forth will completely be eliminated.
  • Agriculture will be so planned that all the people will have enough to consume. Industry will be conducted on a cottage basis till all the people in the village are gainfully employed.
  • The needs of the village will be determined by the people of the village themselves, through the Village Council, representative of the whole village.

Principles of the Sarvodaya:

  • Politics will not be the instrument of power but an agency of service and Rajniti will yield place to Lokniti.
  • All people will be imbued with the spirit of love, fraternity, truth, non-violence and self-sacrifice. Society will function based on non-violence.
  • There will be no party system and majority rule and society will be free from the evil of the tyranny of the majority.
  • The Sarvodaya society is socialist in the true sense of the term. All calling will be the same moral, social and economic values. The individual personality has the fullest scope for development.
  • The Sarvodaya society is based on equality and liberty. There is no room in it for exploitation and class hatred.
  • Sarvodaya stands for the progress of all. All individuals should do individual labour and follow the ideal of nonpossession. Then it will be possible to realize the goal of: each according to his work and each according to his needs.
  • There will be no private property, the instrument of exploitation and the source of social distinctions and hatred.
  • The gain to the individual would be small. The development of each quality depends upon every other. If all the qualities are improved a little, then the individual would gain more.

Sarvodaya Movement:

Gandhi’s ideals have lasted well beyond Indian independence. His followers in India (notably, Vinoba Bhave) continued working to promote the kind of society that he envisioned, and their efforts have come to be known as the Sarvodaya Movement. Anima Bose has referred to the movement’s philosophy as “a fuller and richer concept of people’s democracy than any we have yet known.” Sarvodaya workers associated with Vinoba, J. P. Narayan, Dada Dharmadhikari, Dhirendra Mazumdaar, Shankarrao Deo, K. G. Mashruwala undertook various projects aimed at encouraging popular selforganization during the 1950s and 1960s, including Bhoodan and Gramdan movements. Many groups descended from these networks continue to function locally in India today.

4. Satyagraha

Gandhi called his overall method of nonviolent action Satyagraha . This translates roughly as “ Truth-force.” A fuller rendering, though, would be “the force that is generated through adherence to Truth.” Nowadays, it’s usually called non-violence. But for Gandhi, non-violence was the word for a different, broader concept-namely, “a way of life based on love and compassion .” In Gandhi’s terminology, SatyagrahaTruth-force was an outgrowth of nonviolence.

Satyagraha, when used as a tool for social and political change, aims to win over an opponent. There are three stages in this process:

  • The first stage is that of persuasion through reason.
  • The second stage is characterized by persuasion through suffering. The satyagrahi, at this stage, dramatizes the issues at stake by willingly undergoing self-suffering instead of inflicting suffering on the opponent as a test for the truth element in his cause.
  • If neither persuasion through reason nor self-suffering does succeed to win over the opponent, the satyagrahi resorts to non-violent coercion characterized by tools such as non-cooperation or civil disobedience.

According to Gandhi, “Satyagraha is literally holding on to Truth, and it means therefore Truth-force.” Truth, for Gandhi, was God. Gandhi defined his personal goal as to “seeing God face to face.” Gandhi, in his experiments with satyagraha, both in South Africa and in India, became more and more aware of the relative character of truth as an operative principle. The relative character of truth became evident to Gandhi as each time the social and political problems he took up for reform differed. In this respect, satyagraha is not a dogma. It is neither static nor substantial. For Gandhi, holding on to truth in satyagraha is a dynamic concept and satyagraha is a technique of action.

In satyagraha, self-suffering is willingly accepted by the satyagraha himself with the specific intention of the moral persuasion of the enemy. Self-suffering is neither an inability to win over the opponent through violence nor a meek submission to the will of the evil-doer. It is a fight against an evil system and a tyrant with one’s soul force. In other words, self-suffering is the way of the strong. Gandhi says, “Non-violence cannot be taught to a person who fears to die and has no power of resistance.”

Emphasizing on the inseparableness of truth and nonviolence in satyagraha, Gandhi considers truth and nonviolence (love) as the two sides of the same coin. Ahimsa and Truth are so intertwined that it is practically impossible to disentangle and separate them. Nevertheless, ahimsa is the means; Truth is the end. Means to be means must always be within our reach, and so ahimsa is our supreme duty. If we take care of the means, we are bound to reach the end sooner or later.” In short, in the satyagraha movement for Gandhi, truth is the ultimate goal and non-violence is the means to it.

Satyagraha as a technique for social and political change, has certain definite characteristics and features among which adherence to truth, non-violence and self- suffering have paramount importance.

Frequently Asked Questions (FAQs)

Q: who was mahatma gandhi and why is he important.

A: Mahatma Gandhi, born Mohandas Karamchand Gandhi, was a prominent leader of the Indian independence movement against British colonial rule. He advocated for nonviolent resistance and civil disobedience as powerful tools for social and political change. His principles of truth, nonviolence, and Satyagraha inspired movements for civil rights and freedom across the world.

Q: What were Mahatma Gandhi’s key contributions to India’s independence movement?

A: Gandhi’s contributions to India’s independence movement were profound. He led numerous campaigns against British rule, including the Salt March, the Quit India Movement, and various boycotts. His philosophy of Satyagraha, or nonviolent resistance, mobilized millions of Indians to challenge British authority peacefully, ultimately leading to India’s independence in 1947.

Q: How did Mahatma Gandhi influence the world beyond India’s independence?

A: Gandhi’s influence extended far beyond India’s borders. His philosophy of nonviolence inspired numerous movements for civil rights, social justice, and freedom around the globe. Figures like Martin Luther King Jr., Nelson Mandela, and Cesar Chavez drew inspiration from Gandhi’s teachings and applied them to their own struggles for equality and justice.

Q: What were Mahatma Gandhi’s views on religion and spirituality?

A: Gandhi’s spirituality played a central role in his life and philosophy. He believed in the unity of all religions and emphasized the importance of religious tolerance and understanding. Gandhi’s concept of Sarvodaya, or the welfare of all, was rooted in his belief in the inherent goodness of humanity and the interconnectedness of all beings.

Q: How did Mahatma Gandhi’s legacy continue after his death?

A: Gandhi’s legacy continued to inspire movements for social change and nonviolent resistance long after his death. His principles of truth, nonviolence, and Satyagraha remain relevant in today’s world, influencing activists, leaders, and ordinary people striving for a more just and peaceful society. Gandhi’s teachings serve as a timeless reminder of the power of compassion, courage, and moral integrity in the face of injustice and oppression.

In case you still have your doubts, contact us on 9811333901.  

For UPSC Prelims Resources,  Click here

For Daily Updates and Study Material:

Join our Telegram Channel –  Edukemy for IAS

  • 1. Learn through Videos –  here
  • 2. Be Exam Ready by Practicing Daily MCQs –  here
  • 3. Daily Newsletter – Get all your Current Affairs Covered –  here
  • 4. Mains Answer Writing Practice –  here

Visit our YouTube Channel –  here

  • UPSC Essay Notes – Famous Book Summaries – ARTHASHASTRA
  • UPSC Essay Notes – Famous Book Summaries – THE WORLD IS FLAT
  • UPSC CSE Mains 2021: Paper 1 – Essay Paper
  • Education is what remains after one has forgotten what one has learned in school – UPSC Essay PYQ 2023

' src=

Edukemy Team

Education is what remains after one has forgotten what one..., upsc essay notes – famous book summaries – the art..., upsc essay notes – interesting movies & literature – the..., gandhian philosophy – important personalities, the time to repair the roof is when the sun..., upsc essay 2023 question paper – edukemy, a society that has more justice is a society that..., upsc essay notes – important personalities – lee kuan yew, upsc essay notes – interesting movies & literature – an..., upsc essay notes – interesting movies & literature – coco, leave a comment cancel reply.

Save my name, email, and website in this browser for the next time I comment.

Our website uses cookies to improve your experience. By using our services, you agree to our use of cookies Got it

Keep me signed in until I sign out

Forgot your password?

A new password will be emailed to you.

Have received a new password? Login here

mahatma gandhi essay upsc

mahatma gandhi essay upsc

  • TRP for UPSC Personality Test
  • Interview Mentorship Programme – 2023
  • Daily News & Analysis
  • Daily Current Affairs Quiz
  • Baba’s Explainer
  • Dedicated TLP Portal
  • 60 Day – Rapid Revision (RaRe) Series – 2024
  • English Magazines
  • Hindi Magazines
  • Yojana & Kurukshetra Gist
  • PT20 – Prelims Test Series
  • Gurukul Foundation
  • Gurukul Advanced – Launching Soon
  • Prelims Exclusive Programme (PEP)
  • Prelims Test Series (AIPTS)
  • Integrated Learning Program (ILP) – 2025
  • Connect to Conquer(C2C) 2024
  • TLP Plus – 2024
  • TLP Connect – 2024
  • Public Administration FC – 2024
  • Anthropology Foundation Course
  • Anthropology Optional Test Series
  • Sociology Foundation Course – 2024
  • Sociology Test Series – 2023
  • Geography Optional Foundation Course
  • Geography Optional Test Series – Coming Soon!
  • PSIR Foundation Course
  • PSIR Test Series – Coming Soon
  • ‘Mission ಸಂಕಲ್ಪ’ – Prelims Crash Course
  • CTI (COMMERCIAL TAX INSPECTOR) Test Series & Video Classes
  • Monthly Magazine

Gandhi as Political thinker and a Social reformer

  • October 5, 2021

UPSC Articles

Print Friendly, PDF & Email

Oct 2, 2021: Gandhi Jayanti Special: https://www.youtube.com/watch?v=4hrR7qZFzyU

MODERN HISTORY/ ETHICS

  • GS-1: Indian Freedom Struggle
  • GS-4: Human Values – lessons from the lives and teachings of great leaders.

Context : PM bowed to Mahatma Gandhi on his Jayanti

Mahatma Gandhi entry into to the Indian national movement was a decisive turn towards a broad-based popular struggle. Gandhi’s philosophy was well accepted by both the masses and the nationalist leaders and his political programme was well received and saw wide-spread participation across India.

Born on October 2, 1869, in Porbandar, Gujarat

Gandhi a political thinker and a social reformer

There is more to Gandhi which makes him a political thinker and a relevant social reformer. Gandhi was a dialogical thinker who was open to other horizons of thinking. He firmly believed that the spirit of genuine reciprocity and solidarity is not just a moral requirement, but also a geopolitical necessity.

  • Gandhi rejected the idea that there is one privileged path to god. He also believed that all religious traditions are an unstable mixture of truth and error.
  • He encouraged inter-religious and inter-cultural dialogue, so that individuals could see their faith and culture in a comparative and critical reflection of the other.
  • As such, Gandhi considered interculturalism as a call for simultaneous awareness of commonalities, acceptance of differences, and recognition of shared values.
  • Interestingly, Gandhi was a political thinker and a social practitioner who was constantly experimenting with modes of comparative and cross-border cultural constellations. In Gandhi’s political thinking, the experience of freedom derives not only from constitutional rights but mainly from the diverse modes of participation of the individual in a common humanity. Today, many around the world consider Gandhian ideas as impractical, not to say utopian.
  • Gandhian ethics of social and political reconstruction are more relevant than ever, since they represent an act of self-transformation of humanity rather than an illusory dream of a political leader. Gandhi wanted to change the values that govern the social, political and economic activities in human society.
  • Gandhi believed that decentralised politics and an egalitarian economy function better at the level of micro-communities, where citizens can operate in relations of reciprocity and mutuality. For him, it was clear that neither society nor the individual can live without a moral vision of the world. Gandhi had his moral and political dreams of changing humanity.

Gandhi was a man of experimentation, a man who insisted on the quest for truth. Therefore, it should not come to us as a surprise that the literal meaning of satyagraha is “asserting for truth”.

Reasons for Acceptance of Gandhi’s philosophy and political programme:

  • Demonstrated results in Africa:
  • Gandhiji, by the use of satyagraha and ahimsa as tools was able to secure major demands relating to poll tax, registration certificates etc., from the British government.
  • Tolstoy farm illustrated the peace time utility of ashramas in helping the masses through constructive work and prepare them for popular struggle. 
  • Early successes in India: Through Champaran satyagraha, Ahmedabad mill strike and Kheda satyagraha – he demonstrated the utility of satyagraha and non-violent struggle. 
  • Practical philosophy and political programmes: tools like Satyagraha and ahimsa could have been used by every section of the society especially the masses. The methods like petitions, constitutional struggle hitherto used were not possible to be followed by masses.
  • Belief in masses: 
  • Gandhiji used to say, India live in the villages and it is only through masses the freedom can be achieved. This was not the case with earlier nationalist leaders including moderates and extremists who involved masses on a limited scale.
  • He held all India public meetings focused mainly on the participation of masses.
  • Identification with masses: 
  • Gandhian followed the philosophy of ‘practice what you preach’. For instance, he popularized charkha by using it personally to weave his clothes. He shunned his elite clothes and wore a dhoti to identify himself with the masses.
  • As Ramachandra Guha noted – he dressed like them, walked among them and a sense of belongingness was developed among the masses. Hence, they followed him. 
  • Secular leadership: every strategy and programmes of Gandhiji was secular and he incorporated members of all the religions without any skepticism or discrimination. He took up the issues of all the factions. For instance, he supported Ali brothers in Khilafat movement, supported Akali movement, Temple entry movement etc.,
  • Social issues included in political programmes: 
  • The political programmes of Gandhiji included Dalit upliftment, women emancipation and hence found widespread participation of these sections.
  • Further, the philosophy of Sarvodaya, Antyodaya etc., tried to address the prevailing issues including inequality, rural poverty, food insecurity etc., and hence was widely supported. 
  • Peace time constructive work: 
  • Ashramas provided a way help those who participated in struggle and build momentum garnering public support.
  • Programmes like promotion of Khadi helped Indian producers and hence found support.
  • Establishing local schools provided alternatives to students who left British schools for participating in freedom struggle.
  • Supporting local issues like demand of linguistic provinces, Vaikom satyagraha, Malabar Muslim protest etc., 
  • Effective use of Newspaper and journals: Gandhiji popularized his philosophy through Harijan and the use of local dialects helped in spreading of his message to large number of people especially in rural areas.
  • Home rule movement: under Tilak and Annie Besant prepared a base for Gandhiji demonstrating self-rule which found its resonance in Gandhian philosophy of Swaraj.

Did you Know?

  • Directive Principles of State Policy (DPSPs):  Inspired by Gandhian philosophy; Co-operative societies, women participation and empowerment, socio-economic equality etc,.
  • Decentralization:  Keeping Mahatma Gandhi’s principle of ‘Swarajya’ in mind to strengthen grass root administration.
  • Do or Die Speech by Gandhiji:  In  1942 , Mahatma Gandhi gave the clarion call of ‘Do or Die’ from  Gowalia Tank Maidan  to end the British rule and launched the  Quit India Movement .
  • Who gave the title of ‘Mahatma’ to the ‘Father of the Nation’ : Gurudev – Rabindranath Tagore
  • The first Nationwide Movement:  Rowlatt Satyagraha
  • Quit India  is also called as India August Movement (August Kranti)
  • Majoor Mahajan Sangh:  Gandhi formed the Majoor Mahajan Sangh, an association for workers’ rights. During those days, “Mahajan” was used as a title of respect for elites. Gandhi inverted the social structure by attaching the name “Mahajan” to “Majoor,” or laborers. With that linguistic choice, Gandhi enhanced the pride of workers.
  • Dyerism:  In 1919, the Rowlatt Act enacted by the British government took away the civil rights of Indians. Those who protested peacefully in Jallianwala Bagh faced merciless police firing on the orders of General R Dyer. That cold-blooded assault was described by Mahatma Gandhi as Dyerism. He employed the concept to denote practices of exclusion, including the ostracisation of the Dalits from all spheres of social life in 1919.

Can you answer the following questions?

  • How did Mahatma Gandhi’s experiments and experiences during his political career in South Africa shape the nationalist movement in India? Analyse
  • Throw light on the significance of the thoughts of Mahatma Gandhi in the present times.

For a dedicated peer group, Motivation & Quick updates, Join our official telegram channel – https://t.me/IASbabaOfficialAccount

Subscribe to our YouTube Channel HERE to watch Explainer Videos, Strategy Sessions, Toppers Talks & many more…

mahatma gandhi essay upsc

Related Posts :

Food security, [video] topper’s strategy- rank 62 aparna m b upsc cse 2020 topper, iasbaba’s tlp plus student.

mahatma gandhi essay upsc

  • DAILY CURRENT AFFAIRS IAS | UPSC Prelims and Mains Exam –30th May 2024
  • UPSC Quiz – 2024 : IASbaba’s Daily Current Affairs Quiz 30th May 2024
  • DAILY CURRENT AFFAIRS IAS | UPSC Prelims and Mains Exam –29th May 2024
  • UPSC Quiz – 2024 : IASbaba’s Daily Current Affairs Quiz 29th May 2024
  • DAILY CURRENT AFFAIRS IAS | UPSC Prelims and Mains Exam –28th May 2024
  • UPSC Quiz – 2024 : IASbaba’s Daily Current Affairs Quiz 28th May 2024
  • DAILY CURRENT AFFAIRS IAS | UPSC Prelims and Mains Exam –27th May 2024
  • UPSC Quiz – 2024 : IASbaba’s Daily Current Affairs Quiz 27th May 2024
  • DAILY CURRENT AFFAIRS IAS | UPSC Prelims and Mains Exam –25th May 2024
  • UPSC Quiz – 2024 : IASbaba’s Daily Current Affairs Quiz 25th May 2024

Don’t lose out on any important Post and Update. Learn everyday with Experts!!

Email Address

Search now.....

Sign up to receive regular updates.

Sign Up Now !

mahatma gandhi essay upsc

General Studies

All Programmes

Study Material

Mahatma Gandhi's Movements during Freedom Struggle

Quest for upsc cse panels.

Mahatma Gandhi's Movements during Freedom Struggle-Image

Sub-Categories:

GS-I: Modern History

Prelims : History of India and Indian National Movement.

Mains : Modern Indian History from about the middle of the eighteenth century until the present- significant events, personalities, issues

Mahatma Gandhi Movements: Mohandas Karamchand Gandhi (Mahatma Gandhi) was born on October 2, 1869, in Porbandar, Gujarat. He earned a degree in law from England in 1891. Before entering Indian politics in 1915, he was in South Africa from 1893 to 1914. In the course of his struggle in South Africa, he developed his political philosophy based on non-violence and Satyagraha to give a new direction to the mass movement.

The emergence of Mahatma Gandhi in Indian politics marked the beginning of a new phase in the Indian national movement, the phase of mass movements. This made Gandhi become the most important figure in the history of the Indian freedom struggle.

mahatma gandhi essay upsc

Arrival of Gandhi in India

Gandhi returned to India in January 1915. His efforts were well known in South Africa, not just among the educated but also among the common people.

  • Gandhiji spent a year travelling around British India, getting to know the land and its people on the advice of Gopal Krishna Gokhale . In February 1916, he made his first major public appearance at the inauguration of the Banaras Hindu University (BHU) .
  • Gandhiji’s speech at Banaras revealed that Indian nationalism was an elite phenomenon, and he wished tomake Indian nationalism more properly representative of the Indian people as a whole.

Champaran Satyagraha (1917)

Champaran Satyagraha was the first attempt at mobilising the Indian masses by Gandhi on an invitation by Rajkumar Shukla in the context of indigo peasants of Champaran. This Gandhian Movement was the First Civil Disobedience in India.

  • It was mainly due to the tinkathia system of indigo farmers , where peasants were forced to grow indigo on 3/20 part of their total land. 
  • Gandhi intervened in the matter but was asked to leave the place by authorities. However, Gandhi refused to leave, thus disobeying the order. Eventually, Gandhi was able to convince the government about the illness of the tinkathia system and look into the matter. 
  • The government appointed a committee to go into the matter and nominated Gandhi as a member. As a result, the tinkathia system was abolished, and in a compromise settlement, only 25 % of the money taken by peasants was compensated.
  • Participants: Rajendra Prasad, Narhari Parekh, and J.B. Kripalani 

Ahmedabad Mill Strike (1918)

Following the Champaran Satyagraha, the next step in mobilising the masses was the workers of Ahmedabad's urban centre. Ahmedabad Mill Strike was the result of the disagreement between the textile workers and the mill owners arose in March 1918 regarding the end of the plague bonus. 

  • Mill owners paid bonuses of 75% of their salary to retain the workers during the outbreak of the plague in 1917. But, after the end of the plague, workers were demanding an increase of 50 % in wages while mill owners were advocating discontinuance of the bonus. As a result, a deadlock was created. 
  • Gandhi intervened at the request of Anusuya Sarabhai and undertook a fast unto death. As a result, owners agreed to the 35% wage hike.
  • This was the first movement where fasting , a means of self-suffering to create moral pressure, was used by Gandhi as a political weapon in India.

Kheda Satyagraha (1918)

Due to the failure of the monsoon, the peasants of the Kheda district were in distress. In 1918, they mobilised themselves, demanding revenue relief from the government due to the crop failure and rise in prices. Kheda Satyagraha was the First Non-Cooperation by Gandhi in the Indian National Movement.

  • According to the government's famine code, cultivators were entitled to total remission if crop yield fell below 25% of the average. But the authorities rejected it. As a result, peasants turned to Gandhi.
  • Gandhi intervened on behalf of the poor peasants, advising them to withhold payment and 'fight unto death against such a spirit of vindictiveness and tyranny.'
  • The government ordered it to be restrained in the collection of revenues (collected only from those ryots who could afford to pay) and not to confiscate lands. Gandhi decided to withdraw from the struggle.
  • Participants: Sardar Vallabhbhai Pate l , Narahari Parikh, and Indulal Yagnik.

Satyagraha Against the Rowlatt Act (1919)

In 1917, a sedition committee was formed under Justice Sidney Rowlatt to curb revolutionary activities and investigate the ‘seditious conspiracy’. It recommended the Rowlatt Act (Anarchic and Revolutionary Offenses Act of 1919) ought to limit the liberty of the people passed by the Imperial Legislative Council. Gandhi launched the Satyagraha against the Rowlatt Act . 

  • The Rowlatt Act allowed political and revolutionary activists to be tried without judicial proceedings or even imprisoned without trial for two years. It also suspended the right to habeas corpus and the right to appeal. 
  • Gandhiji called it the “Black Act '' and launched the satyagraha against the Act. It involved fasting, praying, disobeying laws, and risking arrest and imprisonment. The satyagraha brought Gandhi to the centre of the Indian national movement. 
  • Gandhi also organised Satyagraha Sabha, his own organisation, to carry out the movement. However, due to the violence in some parts of India, Gandhi had to call off the Satyagraha. 
  • Gandhi's ability to connect with the masses was demonstrated during this Satyagraha. Gandhian ideology and methods for the freedom struggle (Non-violence and Satyagraha) were introduced to the Indian masses. 

Non-Cooperation Movement (1921-22)

The horrific massacre at Jallianwala Bagh took place in the backdrop of protests against the Rowlatt Act. As a result, Mahatma Gandhi launched the Non-Cooperation Movement in 1920.

  • In September 1920, the Congress held a special session in Calcutta and resolved to accept Gandhi's proposal of non-cooperation with the British government until Khilafat and Punjab grievances were addressed and self-government was established.
  • This Gandhian movement was merged with the Khilafat movement, which demanded that the Turkish Sultan or Khalifa retain control over the Muslim sacred places in the erstwhile Ottoman empire. 
  • Methods: Non-cooperation movement included the boycott of schools, colleges, courts, government offices, legislatures, and foreign goods and the return of government-conferred titles and awards.
  • Withdrawal: Mahatma Gandhi withdrew the non-cooperation movement following the Chauri Chaura violent incident on 5 February 1922 , in which 23 police officers were killed. After the withdrawal, he focussed on the constructive programme of social reforms. 

Civil Disobedience Movement (1930-34)

After the Non-Cooperation Movement, the Civil Disobedience Movement (also known as Salt Satyagraha ) is regarded as the second major mass movement and a significant advancement in broadening the social reach of India's freedom struggle. 

  • Gandhi declared at the Calcutta Congress in 1928 that the British must grant India dominion status, or the country would erupt in a revolution for complete independence. The British paid no attention to this. As a result, INC in its Lahore session (1929) demanded ‘Punra Swaraj’ , and decided to celebrate 26th January as ‘ Independence Day’ . It also declared that a civil disobedience movement would be started under the leadership of Gandhi.
  • Gandhi announced the 'Dandi March' against the unjust tax on salt as part of the movement. On April 6 1930 , he violated the salt regulations, thereby launching the Civil Disobedience Movement.
  • Induced by Gandhi's extraordinary endeavours at Dandi, defiance of the salt laws spread throughout the country. However, it was halted for a period after the Gandhi-Irwin Pact. 
  • When the movement was resumed after the failure of the Second Round Table conference, it began to dwindle towards the end of 1932. It was officially withdrawn in May 1934 .

Quit India Movement (1942)

The Quit India Movement (August Kranti Movement) was the ‘third great wave’ of India’s struggle for freedom, launched on August 8, 1942 , under the leadership of Mahatma Gandhi. This Gandhian Movement was more of a rejection of British rule than a traditional Satyagraha, and it influenced the unprecedented and tumultuous events that occurred in Indian history over the next five years.

  • It was the result of Indian disillusionment with British rule, with the immediate causes being the failure of the Cripps mission and the hardships caused by World War II . 
  • On August 8, 1942, the All India Congress Committee met at Gowalia Tank in Bombay and passed the famous Quit India Resolution. On the same day, Gandhi issued his 'Do or Die' call. It demanded an end to British rule in India with immediate effect, the formation of a provisional government after the war and the declaration of free India .
  • As a result, major leaders of the Indian National Congress were arrested and imprisoned without trial by British officials. However, the protests continued across the country with huge mass participation. 
  • Although it did not immediately achieve its goals, it contributed to the weakening of British rule and paved the way for the independence of India . 

PYQs on Gandhian Movements 

Q)  Many voices had strengthened and enriched the nationalist movement during the Gandhian phase. Elaborate (UPSC Mains 2019)

Q)  Throw light on the significance of the thoughts of Mahatma Gandhi in the present times. (UPSC Mains 2018)

Q)  With reference to the British colonial rule in India, consider the following statements: (UPSC Prelims 2019)

  • Mahatma Gandhi was instrumental in the abolition of the system of ‘indentured labour’.
  • In Lord Chelmsford’s ‘War Conference’, Mahatma Gandhi did not support the resolution on recruiting Indians for World War.
  • Consequent upon the breaking of the Salt Law by the Indian people, the Indian National Congress was declared illegal by the colonial rulers.

Which of the statements given above are correct?

a) 1 and 2 only

b) 1 and 3 only

c) 2 and 3 only

d) 1, 2 and 3

Answer: (b)

Q)  Which one of the following is a very significant aspect of the Champaran Satyagraha? (UPSC Prelims 2018)

a) Active all-India participation of lawyers, students and women in the National Movement.

b) Active involvement of Dalit and Tribal communities of India in the National Movement.

c) Joining of peasant unrest to India’s National Movement.

d) Drastic decrease in the cultivation of plantation crops and commercial crops

Answer: (c)

Question 5: Quit India Movement was launched in response to (UPSC Prelims 2013)

a) Cabinet Mission Plan

b) Cripps Proposals

c) Simon Commission Report

d) Wavell Plan

FAQs on Gandhian Movements

What are the 7 major movements of gandhiji.

The seven major movements of Mahatma Gandhi included the Champaran Movement, the Ahmedabad Mill Strike, the Kheda Movement, the Satyagraha against the Rowlatt Act, the Non-cooperation Movement, the Civil Disobedience Movement, and the Quit India Movement.

What was Gandhi's first movement in India?

The Champaran Satyagraha of 1917 was Gandhi's first Satyagraha movement in India and is regarded as a historically significant revolt in the Indian Independence Movement. During the British colonial period, a farmer's uprising occurred in the Champaran district of Bihar, India.

What are some of the challenges that Gandhian movements faced?

Gandhian movements faced challenges such as resistance from colonial authorities, lack of widespread support, internal divisions, and the difficulty of maintaining nonviolent discipline among participants. Additionally, economic and social realities often clashed with the idealistic principles of Gandhi's philosophy, making it challenging to achieve lasting change.

What was the impact of the Gandhian movements on the Indian independence struggle?

The Gandhian movements had a profound impact on the Indian independence struggle. Mahatma Gandhi's philosophy of nonviolent resistance (Satyagraha) and civil disobedience mobilised millions of Indians, uniting them in a peaceful but determined quest for freedom. His leadership inspired the Quit India Movement and other campaigns that eventually led to India gaining independence from British colonial rule in 1947, marking a historic achievement in the nation's history.

Where and when did Mahatma Gandhi make his first public appearance in India upon coming back from South Africa?

On February 4, 1916, in BHU, Gandhiji made his first public appearance since his return from South Africa. He spoke to the crowd in BHU, which was primarily made up of impressionable youngsters, princes, well-dressed individuals, etc.

© 2024 Vajiram & Ravi. All rights reserved

Essay on Mahatma Gandhi – Contributions and Legacy of Mahatma Gandhi

500+ words essay on mahatma gandhi.

Essay on Mahatma Gandhi – Mahatma Gandhi was a great patriotic Indian, if not the greatest. He was a man of an unbelievably great personality. He certainly does not need anyone like me praising him. Furthermore, his efforts for Indian independence are unparalleled. Most noteworthy, there would have been a significant delay in independence without him. Consequently, the British because of his pressure left India in 1947. In this essay on Mahatma Gandhi, we will see his contribution and legacy.

Essay on Mahatma Gandhi

Contributions of Mahatma Gandhi

First of all, Mahatma Gandhi was a notable public figure. His role in social and political reform was instrumental. Above all, he rid the society of these social evils. Hence, many oppressed people felt great relief because of his efforts. Gandhi became a famous international figure because of these efforts. Furthermore, he became the topic of discussion in many international media outlets.

Mahatma Gandhi made significant contributions to environmental sustainability. Most noteworthy, he said that each person should consume according to his needs. The main question that he raised was “How much should a person consume?”. Gandhi certainly put forward this question.

Furthermore, this model of sustainability by Gandhi holds huge relevance in current India. This is because currently, India has a very high population . There has been the promotion of renewable energy and small-scale irrigation systems. This was due to Gandhiji’s campaigns against excessive industrial development.

Mahatma Gandhi’s philosophy of non-violence is probably his most important contribution. This philosophy of non-violence is known as Ahimsa. Most noteworthy, Gandhiji’s aim was to seek independence without violence. He decided to quit the Non-cooperation movement after the Chauri-Chaura incident . This was due to the violence at the Chauri Chaura incident. Consequently, many became upset at this decision. However, Gandhi was relentless in his philosophy of Ahimsa.

Secularism is yet another contribution of Gandhi. His belief was that no religion should have a monopoly on the truth. Mahatma Gandhi certainly encouraged friendship between different religions.

Get the huge list of more than 500 Essay Topics and Ideas

Legacy of Mahatma Gandhi

Mahatma Gandhi has influenced many international leaders around the world. His struggle certainly became an inspiration for leaders. Such leaders are Martin Luther King Jr., James Beve, and James Lawson. Furthermore, Gandhi influenced Nelson Mandela for his freedom struggle. Also, Lanza del Vasto came to India to live with Gandhi.

mahatma gandhi essay upsc

The awards given to Mahatma Gandhi are too many to discuss. Probably only a few nations remain which have not awarded Mahatma Gandhi.

In conclusion, Mahatma Gandhi was one of the greatest political icons ever. Most noteworthy, Indians revere by describing him as the “father of the nation”. His name will certainly remain immortal for all generations.

Essay Topics on Famous Leaders

  • Mahatma Gandhi
  • APJ Abdul Kalam
  • Jawaharlal Nehru
  • Swami Vivekananda
  • Mother Teresa
  • Rabindranath Tagore
  • Sardar Vallabhbhai Patel
  • Subhash Chandra Bose
  • Abraham Lincoln
  • Martin Luther King

FAQs on Mahatma Gandhi

Q.1 Why Mahatma Gandhi decided to stop Non-cooperation movement?

A.1 Mahatma Gandhi decided to stop the Non-cooperation movement. This was due to the infamous Chauri-Chaura incident. There was significant violence at this incident. Furthermore, Gandhiji was strictly against any kind of violence.

Q.2 Name any two leaders influenced by Mahatma Gandhi?

A.2 Two leaders influenced by Mahatma Gandhi are Martin Luther King Jr and Nelson Mandela.

Customize your course in 30 seconds

Which class are you in.

tutor

  • Travelling Essay
  • Picnic Essay
  • Our Country Essay
  • My Parents Essay
  • Essay on Favourite Personality
  • Essay on Memorable Day of My Life
  • Essay on Knowledge is Power
  • Essay on Gurpurab
  • Essay on My Favourite Season
  • Essay on Types of Sports

Leave a Reply Cancel reply

Your email address will not be published. Required fields are marked *

Download the App

Google Play

Optimize IAS

  • Daily Practice Sheets
  • Daily Prelims Notes
  • Prelims Power Play
  • Mains Factly
  • Sunday Essay Sadhna
  • Mains Master Notes
  • Daily Answer Writing
  • Essay Master Class
  • Ethics Master Class
  • Laqshya 2025 Prelims
  • Laqshya 2025 CA
  • OPTIMA NOTES & CARDS
  • OPTIMA SECTIONAL & FLT Tests
  • ARJUNA PRIME 2025
  • Important Topics List for Prelims 2024
  • Daily Prelims Notes Compilation
  • Daily Practice Sheet Compilation
  • PPP Compilation
  • General Studies Notes
  • UPSC Mains Previous Year Papers
  • Portal Login

Contribution of Mahatma Gandhi in freedom movement

  • October 2, 2022
  • Posted by: OptimizeIAS Team
  • Category: DPN Topics

Subject :History

Gandhi Jayanti is celebrated all over the country on October 2.

  • Mahatma Gandhi was born on 2 October, 1869 at Porbandar, Gujarat.
  • Mahatma Gandhi was a  renowned freedom activist  who had played an important role in India’s struggle for Independence against the British rule of India.
  • His ideology of truth and non-violence influenced  many and was also  adopted by Martin Luther and Nelson Mandela for their struggle movement.

Early Life:

  • At the age of 19, Mohandas left home to study law in London at the Inner Temple, one of the city’s four law colleges. Upon returning to India in mid-1891, he set up a law practice in Bombay, but met with little success. He soon accepted a position with an Indian firm that sent him to its office in South Africa. Along with his wife, Kasturbai, and their children, Gandhi remained in South Africa for nearly 20 years.
  • Books that inspired Mahatma Gandhi : Unto this Last by John Ruskin and The Kingdom of God is within you’ by Leo Tolstoy.
  • English artist John Ruskin’s book  Unto This Last  inspired Gandhi and he set up Phoenix Farm near Durban. Here, Gandhi would train his cadres on non-violent Satyagraha or peaceful restraint. Phoenix Farm is considered as the birthplace of Satyagraha. However, it was at the Tolstoy Farm , Gandhi’s second camp in South Africa, where Satyagraha was molded into a weapon of protest.

Associations by Gandhiji in South Africa:

  • Natal Indian Congress
  • Passive Resistance Association

Journals by Gandhiji in South Africa:

  • Indian Opinion

Gandhiji in India:

  • In 1915, after returning from South Africa, where he had perfected the art of non-violent resistance or satyagraha,  Mahatma Gandhi spent the next few years in fully understanding Indian conditions and  travelled widely across the length and breadth of this vast nation .
  • Gandhi also met the Congress leadership and took everyone’s suggestions on board, before taking tentative steps towards launching himself into the Indian Independence struggle.
  • While the Indian freedom movement can be thought of as one single struggle that lasted decades, in reality there were  phases of great activity and relatively lull periods as well.  And much of this calendar of protests and tactical retreat was decided by Gandhi himself, who apart from being the greatest advocate of peace and violence in modern times,  was also a brilliant organiser of mass movements.  He understood the people’s pulse like few others.
  • Champaran Movement:  The Champaran Movement is regarded as the  first modern civil disobedience movement in India . It took place in the then Champaran district of northern Bihar. The  Indian labourers and farm-workers here tilled the land but all the profits went to the European landowners.  The labourers protested but it was Gandhi’s involvement in their struggle that culminated in the Champaran Agrarian Act, 1918, which helped farmers secure greater rights over their own land. The success of Champaran made many more Indians aware of Gandhi and his principles, and the Congress party found its greatest mass leader.
  • Ahmedabad Mill Worker Satyagraha: In March 1918, under the leadership of Gandhi, there was a strike in the cotton mills. In this strike Gandhi used the weapon of Hunger strike.
  • Kheda Satyagraha:  In Kheda, Gujarat, despite crop failures, the  farmers’ desperate pleas for tax remission fell on deaf ears.  Gandhi’s message to them was to withhold revenue and fight peacefully but bravely against such vindictiveness and tyranny. Another rising star of the freedom movement, SardarVallabhbhai Patel , also played a key role in this struggle of 1918. The local government eventually came out with a solution that was acceptable to both parties. The Champaran and Kheda campaigns were limited to specific areas, but they gave Gandhi the confidence to launch his major pan-Indian movements in future.
  • Rowlatt Act Satyagraha : During World War I (1914–18), the British government of India enacted a series of repressive emergency powers that were intended to combat subversive activities. The Anarchical and Revolutionary Crimes Act of 1919 popularly known as Rowlatt Act (Black Act) which was passed on 10 th March, 1919, authorised the government to imprison or confine, without a trial, any person associated with seditious activities which led to nationwide unrest. Gandhiji called for a one-day general strike ( Rowlatt Satyagraha ) throughout the country.
  • Non-Cooperation Movement : The Non-Cooperation movement (1920-22) was the  first mass movement launched by Gandhi, seeking self-government or swaraj  for all Indians. It followed from Gandhi’s deeply held ideals of satyagraha and civil disobedience, and he called upon Indians to boycott all institutions linked to the British including courts and colleges, give up titles and refuse to pay taxes. Audacious in scope, the Non-Cooperation movement may not have been a 100 per cent success, but it made millions of Indians understand the true meaning of a modern, organised political movement and its power.
  • Dandi March : An unqualified masterstroke, the Dandi March brought Mahatma Gandhi’s  political genius and sense of timing to the fore . He started the historic march from Sabarmati Ashram to the coastal village of Dandi in March 1930. While the basic reason was to protest against the unacceptably high salt tax levied by the British, it turned into something much bigger as thousands of people joined Gandhi on his 24-day march. The Dandi March became the talking point across the country and the whole nation was inspired. From that moment onwards,  non-violent resistance against the British became the natural course of action for a vast section of Indians for the remaining years of the Raj. 
  • Quit India Movement:  By the beginning of the 1940s, the British knew that their days in India were numbered, but they used the excuse of World War 2 to delay any talk of India’s independence. In August 1942, the All-India Congress Committee passed the famous ‘Quit India’ resolution in Bombay, under the leadership of Mahatma Gandhi, who also gave the  slogan of ‘Do or Die’ . The entire leadership of the Congress was arrested, but that didn’t stop thousands of protests against British rule in every corner of the country. There was no middle path now: the British had to quit India.

Journals of Gandhiji

  • Mahatma Gandhi was offered editorship of Young India and Navjivan
  • In February 1933 Gandhiji started  Harijan, Harijanbandhu, Harijansevak in English, Gujarati and Hindi,

Organisations by Mahatma Gandhi

  • Harijan Sevak Sangh
  • All India spinners association
  • Ahmedabad Textile Labour Association
  • All India Village Industries’ Association

Examples

The Role of Mahatma Gandhi in India’s Independence

Ai generator.

Good morning, respected teachers, parents, and my dear friends!

Today, we gather to celebrate Indian Independence Day , a day that marks our nation’s freedom from British rule. As we reflect on this historic occasion, it is essential to remember the pivotal role played by one of our greatest leaders— Mahatma Gandhi . His leadership, principles, and unwavering commitment to non-violence were instrumental in securing India’s independence.

Introduction to Mahatma Gandhi

Mohandas Karamchand Gandhi , fondly known as Mahatma Gandhi, was born on October 2, 1869. He is revered as the “Father of the Nation” for his significant contributions to India’s freedom struggle. Gandhi’s philosophy of non-violence (ahimsa) and civil disobedience (satyagraha) became the cornerstone of the Indian independence movement.

Early Life and Influences

Gandhi studied law in England and later worked in South Africa, where he experienced racial discrimination firsthand. These experiences shaped his beliefs and strategies in fighting injustice. He returned to India in 1915 and soon became involved in the struggle for independence.

Key Movements Led by Gandhi

The non-cooperation movement (1920-1922).

Gandhi launched the Non-Cooperation Movement in response to the Jallianwala Bagh massacre and other atrocities committed by the British. He urged Indians to withdraw their cooperation from the British government by boycotting British goods, institutions, and services. This movement united millions of Indians and marked a significant step towards self-rule.

The Salt March (1930)

One of Gandhi’s most iconic acts of civil disobedience was the Salt March in 1930. To protest the British monopoly on salt production and sales, Gandhi led a 240-mile march to the Arabian Sea, where he made salt from seawater. This act of defiance inspired millions of Indians to join the fight against British rule and highlighted the power of non-violent resistance.

The Quit India Movement (1942)

In 1942, Gandhi launched the Quit India Movement , demanding an end to British rule. He called for mass civil disobedience, urging Indians to “Do or Die” in their quest for freedom. The British responded with arrests and violence, but the movement intensified the call for independence and demonstrated the Indian people’s resolve to achieve self-rule.

Gandhi’s Philosophy of Non-Violence

Gandhi’s commitment to non-violence was central to his approach. He believed that non-violent resistance was not only morally superior but also more effective in the long run. His principles of truth (satya) and non-violence inspired countless Indians to join the independence movement and adopt peaceful methods of protest.

Legacy and Impact

Gandhi’s leadership and philosophy left an indelible mark on India and the world. His methods influenced numerous global leaders and movements, including Martin Luther King Jr. and the American Civil Rights Movement. Gandhi’s vision for an independent India was not just political freedom but also social and economic justice for all.

As we celebrate Independence Day, let us remember the immense contributions of Mahatma Gandhi. His unwavering commitment to non-violence, truth, and justice paved the way for India’s freedom. Gandhi’s legacy continues to inspire us to fight for our rights and uphold the values of peace and equality.

Thank you for your attention, and Jai Hind!

Twitter

Text prompt

  • Instructive
  • Professional

10 Examples of Public speaking

20 Examples of Gas lighting

  • IAS Preparation
  • UPSC Preparation Strategy

Mahatma Gandhi National Rural Employment Guarantee Act (MGNREGA)

Latest Update about MGNREGA UPSC

In March 2023, the government declared new wage rates for unskilled manual workers under the Mahatma Gandhi National Rural Employment Guarantee Act, 2005.

MGNREGA UPSC Notes Download PDF Here

  • For the financial year 2023-24, under sub-section(1) of Section 6 of the Mahatma Gandhi National Rural Employment Guarantee Act, 2005, the Ministry of Rural Development has notified the new wage rates. It will become effective from 1st April 2023.
  • Changes in the Consumer Price Index – Agriculture Labour (CPI-AL) is used by the Ministry of Rural Development to revise the wages under MGNREGA.

Recent changes introduced in wages:

  • Wage hike ranges from Rs. 7 to Rs. 26 per day, which is an increase of 2% to 10%.
  • As per the wage revisions carried out, the highest rate of wage, which is Rs. 357 per day has been fixed for Haryana and the lowest wage of Rs. 221 per day has been fixed for Madhya Pradesh and Chhattisgarh.
  • The highest percentage increase has been recorded for Rajasthan, where wages rose from Rs. 231 per day in 2022-23 to Rs. 255 per day for 2023-24, making a 10.39% hike. 
  • The lowest percentage hike is reported in Goa, where wages increased from Rs. 315 per day in 2022-23 to Rs. 322 per day in 2023-24.

The Union Government has made it mandatory for payments to be made for MGNREGA beneficiaries through the Aadhar-Based Payment System or ABPS. Know more about this development in the link, payment system for MGNREGA .

The Union government has allocated Rs.72000 crores for The Mahatma Gandhi National Rural Employment Guarantee Act, (MGNREGA) scheme in 2021-22. Candidates should know about Mahatma Gandhi NREGA as it holds importance not only while you are an IAS Exam aspirant but also when you become an administrator, knowledge of this scheme will be helpful.

Quick Facts about MGNREGA UPSC:

MGNREGA & Workers Crisis – COVID Pandemic

Finance Minister Nirmala Sitharaman on March 26th, 2020, workers under the MGNREGA would get a hike of Rs. 2000 each on average. It was also announced that three crore senior citizens, persons with disabilities, and widows will get a one-time additional amount of Rs 1,000 in two instalments which will be provided through DBT (Direct Benefit Transfer) over three months. This announcement was made as an initiative towards the loss caused by the Covid-19 outbreak. The 21-day lockdown was expected to cost the Indian Economy a cost of around 9 lakh crores.  Funds worth Rs 31,000 crore are also to be provided to augment medical testing, screening, and providing better healthcare facilities to those who have been affected financially due to the COVID-19 outbreak.

It is an important topic for the General Studies Syllabus of the UPSC for Civil Services Exam 2023. This government scheme is covered under Economic and Social Development for Prelims and Economy for Mains.

Enhance your UPSC 2024  preparation today!

To practice questions on Economy and Government Schemes for Prelims, check out:

MGNREGA History:

In 1991, the P.V Narashima Rao government proposed a pilot scheme for generating employment in rural areas with the following goals:

  • Employment Generation for agricultural labour during the lean season.
  • Infrastructure Development
  • Enhanced Food Security

This scheme was called the Employment Assurance Scheme which later evolved into the MGNREGA after the merger with the Food for Work Programme in the early 2000s.

Objectives of MGNREGA:

The Mahatma Gandhi National Rural Employment Guarantee Act (MGNREGA) has the following objectives:

  • Provide 100 days of guaranteed wage employment to rural unskilled labour
  • Increase economic security
  • Decrease migration of labour from rural to urban areas

MGNREGA differentiates itself from earlier welfare schemes by taking a grassroots-driven approach to employment generation. The programs under the act are demand-driven and provide legal provisions for appeal in the case, work is not provided or payments are delayed. The scheme is funded by the central government which bears the full cost of unskilled labour and 75% of the cost of material for works undertaken under this law. The central and state governments audit the works undertaken under this act through annual reports prepared by the CEGC (Central Employment Guarantee Council) and the SEGC (State Employment Guarantee Councils). These reports have to be presented by the incumbent government in the legislature.

A few salient features of the scheme are:

  • It gives a significant amount of control to the Gram Panchayats for managing public works, and strengthening Panchayati Raj Institutions. Gram Sabhas are free to accept or reject recommendations from Intermediate and District Panchayats.
  • It incorporates accountability in its operational guidelines and ensures compliance and transparency at all levels.

Ever since the scheme was implemented, the number of jobs has increased by 240% in the past 10 years. The scheme has been successful in enhancing economic empowerment in rural India and helping overcome the exploitation of labour. The scheme has also diminished wage volatility and the gender pay gap in labour. This can be substantiated by the following data available at the official site of MGNREGA:

  • 14.88 crores MGNREGA job cards have been issued (Active Job Cards – 9.3 crores)
  • 28.83 crores workers who gained employment under MGNREGA (2020-21) out of which active workers are 14.49 crores.

What is the role of Gram Sabha and Gram Panchayat in MGNREGS?

The role of Gram Sabha in the Mahatma Gandhi National Rural Employment Guarantee Scheme is tabled below:

The role of Gram Panchayat in the Mahatma Gandhi National Rural Employment Guarantee Scheme is tabled below:

Role of State Governments in MGNREGS

The important roles of the state government in executing the MGNREGA scheme are:

  • It frames rules charting out the state’s responsibility under the act.
  • It sets up the State Employment Guarantee Council.
  • State Employment Guarantee Fund (SEGF) is established by state governments.
  • It makes sure to dedicate the Employment Guarantee Assistant (Gram Rozgar Sahayak), the PO and the staff at the State, district, cluster and Gram Panchayat level; for the execution of the scheme.

MGNREGA – State Employment Guarantee Council (SEGC)

The State Employment Guarantee Council is responsible for advising the state government for the implementation of the MGNREG scheme. Some important functions of SEGC under MNREGS are:

  • The suggestion of improvements in the execution of the scheme.
  • Evaluation and monitoring of the scheme.
  • To recommend proposals of the works to the central government.
  • To appraise the districts about the scheme and its features.
  • To prepare an annual report to be submitted by the state government before the state legislature.

This law and the employment guarantee schemes which are part of its provisions are important from the IAS exam point of view. UPSC aspirants should read about this government scheme in detail as questions related to this topic are asked in the Prelims exam and in General Studies paper II. The questions for government schemes like MGNREGA are classified under Welfare schemes, and the topic has a significant overlap with topics like human development, poverty, and hunger.

Technocratic Subversion of MGNREGA

In recent months, MGNREGA employees from all over the nation have been peacefully protesting as members of the NREGA Sangharsh Morcha, a national alliance of labour unions. A multitude of complaints, including late wage payments, unfavourable working conditions, and insufficient MGNREGA scheme execution, are to blame for the protests.

News in detail: 

Mahatma Gandhi National Rural Employment Guarantee Act (MGNREGA) workers demand

  • adequate funds for work and timely wages, 
  • release of pending funds to West Bengal, 
  • revocation of mandatory attendance using the National Mobile Monitoring System (NMMS) app , and
  • revocation of wage payments using the Aadhaar-based payment systems (ABPS).

Read more on the MGNREGA payment system in the linked article.

Way forward

  • The majority of Indians cannot perceive and evaluate how their government is acting since the state has placed so much emphasis on “seeing” its citizens.
  • According to democratic dogma, individuals should be able to clearly “see” the state, not the other way around. If not, people’s respect for and confidence in them are at risk.
  • It’s important to handle the MGNREGA corruption cases. But for that, social audits must be strengthened rather than adding to the struggles of already overworked women.
  • It is important to examine the causes of ineffective fund management and take action to improve it.
  • Additionally, officers who are found guilty of misusing funds should face legal action.
  • By increasing awareness and making it more inclusive, the participation of women and members of underprivileged classes must increase.

Frequently Asked Questions Mahatma Gandhi National Rural Employment Guarantee Act (MGNREGA)

What is the aim of mgnrega, what is the objective of mgnrega.

To read more about UPSC preparation, please visit:

Daily News

Leave a Comment Cancel reply

Your Mobile number and Email id will not be published. Required fields are marked *

Request OTP on Voice Call

Post My Comment

mahatma gandhi essay upsc

IAS 2024 - Your dream can come true!

Download the ultimate guide to upsc cse preparation, register with byju's & download free pdfs, register with byju's & watch live videos.

COMMENTS

  1. Mahatma Gandhi: The Father of the Nation

    Mahatma Gandhi was the greatest leader of the Indian independence movement. His life was his lesson, and inspired many around the world. ... ClearIAS is one of the most trusted learning platforms in India for UPSC preparation. Around 1 million aspirants learn from the ClearIAS every month. Our courses and training methods are different from ...

  2. Mahatma Gandhi

    This topic of "Mahatma Gandhi - Biography, Movements, Literary Works" is important from the perspective of the UPSC IAS Examination, which falls under General Studies Portion. Early Life of Mahatma Gandhi: Mohandas Karamchand Gandhi was born October 2, 1869, in Porbandar, India, to Karamchand Gandhi and Putlibai.

  3. Essay on Mahatma Gandhi for Students in English [500+ Words]

    Essay on Mahatma Gandhi in English. Mahatma Gandhi was a man of power, honesty, sacrifices, and simplicity. He was a patriotic personality. His life was unbelievable. He never liked others' appreciation. His efforts to protect India are unparalleled. Only with his efforts, the British left India in 1947. Mohandas Gandhi is another name of ...

  4. Gandhian Ideologies

    What is Gandhian ideology? Gandhian ideology is the set of religious and social ideas adopted and developed by Mahatma Gandhi, first during his period in South Africa from 1893 to 1914, and later in India. Gandhian philosophy is not only simultaneously political, moral and religious, it is also traditional and modern, simple and complex.

  5. Gandhian Philosophy

    2. Satyagraha. The concept was introduced in the early 20th century by Mahatma Gandhi and designated a determined but nonviolent resistance to evil. The supreme idea of truth naturally leads to the Gandhian ideology of Satyagraha. Protecting the standards and tenets of truth is essential if it is the ultimate reality.

  6. UPSC Essay Notes

    1. Nai Talim. In 1937, Mahatma Gandhi seeded an important idea to revamp the education system, at a conference in Wardha,B Maharashtra through Nai Talim. Gandhi felt a need to nullify what education at the time of British Raj was trying to create - distinctions between knowledge and work, teaching and learning, among others.

  7. RSTV IAS UPSC

    India and Mahatma Gandhi Archives TOPIC: Paper I: Essay General Studies 1: Personalities in Indian national movements General Studies 4: Ethics; Indian thinkers and philosophers Mahatma Gandhi: Born on October 2, 1869, in Porbandar, Gujarat Gandhi: Philosophy and significance in present times Opinions and views about his person and his non-violent technique of…

  8. Gandhi's Early Movements In India [Modern History Of India Notes For UPSC]

    With the arrival of Mahatma Gandhi in the Indian Independence struggle, there have been led, many significant movements which are important to be read for IAS Exam. The famous satyagraha movement includes - Champaran Satyagraha, Ahmedabad Mill Strike and Kheda Satyagraha. This article will provide you with NCERT notes that are useful for ...

  9. Ideals of Mahatma Gandhi

    The ideals of truth and nonviolence, which underpin the whole philosophy, are relevant to all humankind, and are considered as universal by the Gandhians. More than ever before, Mahatma Gandhi's teachings are valid today, when people are trying to find solutions to the rampant greed, widespread violence, and runaway consumptive style of living.

  10. Gandhi as Political thinker and a Social reformer

    Mahatma Gandhi entry into to the Indian national movement was a decisive turn towards a broad-based popular struggle. Gandhi's philosophy was well accepted by both the masses and the nationalist leaders and his political programme was well received and saw wide-spread participation across India. Born on October 2, 1869, in Porbandar, Gujarat ...

  11. Free Answer Writing Practice Question For IAS Mains Exam 2021

    All GS Paper 1 GS Paper 2 GS Paper 3 GS Paper 4 Essay Revision Tests. 02 Dec 2021 GS Paper 4 Theoretical Questions. Q. Mahatma Gandhi created a moral system which also served as the framework for his political practice. Elucidate. (150 Words)

  12. Mahatma Gandhi Movements during Indian Freedom Struggle

    Mahatma Gandhi Movements: Mohandas Karamchand Gandhi (Mahatma Gandhi) was born on October 2, 1869, in Porbandar, Gujarat. He earned a degree in law from England in 1891. Before entering Indian politics in 1915, he was in South Africa from 1893 to 1914. In the course of his struggle in South Africa, he developed his political philosophy based on non-violence and Satyagraha to give a new ...

  13. Role of Mahatma Gandhi in Freedom Struggle of India

    Role of Mahatma Gandhi in Freedom Struggle UPSC. Mahatma Gandhi is the most persuasive leader and an esteemed individual who participated in India's struggle for independence from British Raj. He was an anti-colonialist, non-violent freedom fighter of India who led the country towards sovereignty without the use of any weapon. He believed in ...

  14. Rise of Gandhi in Indian Freedom Struggle: NCERT Notes For UPSC

    An all India level mass protest was launched by Gandhi on April 6, 1919 against the Rowlatt act. Satyagraha was introduced to masses by Gandhi and it assumed a national character. It was the first mass strike and the biggest upsurge against the British since 1857. Check NCERT Notes on Government of India Act 1919 here.

  15. Essay on Mahatma Gandhi

    500+ Words Essay on Mahatma Gandhi. Essay on Mahatma Gandhi - Mahatma Gandhi was a great patriotic Indian, if not the greatest. He was a man of an unbelievably great personality. He certainly does not need anyone like me praising him. Furthermore, his efforts for Indian independence are unparalleled. Most noteworthy, there would have been a ...

  16. Philosophy Of Mahatma Gandhi: Exclusive Notes To Ace UPSC Exam!

    The philosophy of Mahatma Gandhi was first developed in South Africa from 1893 to 1914 and then in India. The philosophy of Mahatma Gandhi is political, moral, and religious at the same time. It is traditional and modern, straightforward and complex at the same time. Gandhiji was exposed to many Western influences.

  17. Relevance of Gandhian Principles in the 21st Century

    In today's fast-paced and ever-changing world, the relevance of timeless wisdom often gets lost in the shuffle. However, the principles advocated by Mahatma Gandhi, also known as the Father of the Indian Nation, continue to shine as beacons of hope and wisdom in the 21st century. With simplicity, non-violence, truthfulness, self-reliance, and compassion at their core, Gandhian principles offer ...

  18. Indian National Movement and Mahatma Gandhi

    The non-cooperation movement was a nationwide movement launched on September 04, 1920 under the leadership of Mahatma Gandhi. The movement was launched in the wake of a series of events like the Jallianwala Bagh massacre and the passing of the Rowlatt Act. The movement was able to accrue the sympathy of the masses and established Gandhiji as ...

  19. Contribution of Mahatma Gandhi in freedom movement

    Gandhi Jayanti is celebrated all over the country on October 2. Mahatma Gandhi was born on 2 October, 1869 at Porbandar, Gujarat. Mahatma Gandhi was a renowned freedom activist who had played an important role in India's struggle for Independence against the British rule of India. His ideology of truth and non-violence influenced many and was ...

  20. The Role of Mahatma Gandhi in India's Independence

    Introduction to Mahatma Gandhi. Mohandas Karamchand Gandhi, fondly known as Mahatma Gandhi, was born on October 2, 1869. He is revered as the "Father of the Nation" for his significant contributions to India's freedom struggle. Gandhi's philosophy of non-violence (ahimsa) and civil disobedience (satyagraha) became the cornerstone of the ...

  21. Gandhism: Ideology and Relevance in 21st Century

    Gandhian ideology (also known as Gandhism) is the set of religious and social ideas adopted and developed by India's Father of Nation, Mahatma Gandhi, first during his period in South Africa from 1893 to 1914, and later in India. Although incorporating certain Western ideologies that Mahatma Gandhi was exposed to, Gandhian ideologies were ...

  22. Relevance of Gandhism Today

    Relevance of Gandhism Today. This article is based on "Thinking of Gandhi Today" which was published in the Economic and Political weekly on 19/09/2020. It talks about the relevance of Gandhism in present times. 2 nd October every year is a great day for India as well as for the world. On this day India celebrates the birth anniversary of ...

  23. Mahatma Gandhi National Rural Employment Guarantee Act (MGNREGA)

    MGNREGA is an important government scheme in the UPSC 2024 Exam Syllabus. Read MGNREGS objectives, the role of the state, gram sabha, gram panchayat in MGNREGA. Get important facts of Mahatma Gandhi NREGA for IAS exam preparation.

  24. Main Answer Writing Practice

    Satyagraha, which means "truth force" or "soul force," was a method of non-violent resistance that Gandhi developed and used extensively in India's independence struggle. It involved the use of civil disobedience, strikes, boycotts, and other non-violent means to challenge unjust laws and oppressive policies. Satyagraha aimed at converting the ...